v=u + 2at
Where v is the final velocity (in m/s), u is the initial velocity (in m/s), a is the
acceleration (in m/s?) and t is the time in seconds).
Find v when u is 35 m/s, a is 28 m/s2, and t is 58 seconds.

Answers

Answer 1

Answer:

3283m/s

Step-by-step explanation:

V=U+2at

V=35+2(28)(58)

V=35+3248

V=3283m/s


Related Questions

32 x square Y - 2 y cube

Answers

Answer:

207y

Step-by-step explanation:

what is the relation that represents the relation

Answers

Answer:

what can i help u with

Step-by-step explanation:

I really can't help u with that sorry i am bad at math

Write an equation of the graph (shown below) in slope intercept form.

Answers

This is not a valid question and, therefore, cannot be answered. (Reason: No graph is attached.) Try revising it or giving more information. Hope this helps! ❤️

express the ratio 7day to 6weeks as a decimal fraction

Answers

Answer:

6 weeks=6*7 days=42days

7/42 =1/6 =0.16667

OR

7 days=1 week

therefore 1/6=0.16667

Note that both must be in the same unit.

Determine, to one decimal place, the length, width & height of the rectangular prism that would have the greatest volume, with a surface area of 200 cm^2.

Answers

Answer:

The length = The width = The height  ≈ 5.8 cm

Step-by-step explanation:

The volume of a rectangular pyramid, V = l × w × h

The surface area of the pyramid = 2 × l × h + 2 × w × h + 2 × l × w = 200

∴  l × h + w × h + l × w = 200/2 = 100

We have that the maximum volume is given when the length, width, and height are equal and one length is not a fraction of the other. Therefore, we get;

At maximum volume, l = w = h

∴ l × h + w × h + l × w = 3·l² = 100

l² = 100/3

l = 10/√3

Therefore, the volume, v = l³ = (10/√3)³

The length = The width = The height = 10/√3 cm ≈ 5.8 cm

for the function g(x)=3-8(1/4)^2-x

a) State the y-intercept

b) State the equation of the horizontal asymptote

c) State whether the function is increasing or decreasing.

d) State the domain and range
e) Sketch the graph

Could anyone help?

Answers

Using function concepts, it is found that:

a) The y-intercept is y = 2.5.b) The horizontal asymptote is x = 3.c) The function is decreasing.d) The domain is [tex](-\infty,\infty)[/tex] and the range is [tex](-\infty,3)[/tex].e) The graph is given at the end of the answer.

------------------------------------

The given function is:

[tex]g(x) = 3 - 8\left(\frac{1}{4}\right)^{2-x}[/tex]

------------------------------------

Question a:

The y-intercept is g(0), thus:

[tex]g(0) = 3 - 8\left(\frac{1}{4}\right)^{2-0} = 3 - 8\left(\frac{1}{4}\right)^{2} = 3 - \frac{8}{16} = 3 - 0.5 = 2.5[/tex]

The y-intercept is y = 2.5.

------------------------------------

Question b:

The horizontal asymptote is the limit of the function when x goes to infinity, if it exists.

[tex]\lim_{x \rightarrow -\infty} g(x) = \lim_{x \rightarrow -\infty} 3 - 8\left(\frac{1}{4}\right)^{2-x} = 3 - 8\left(\frac{1}{4}\right)^{2+\infty} = 3 - 8\left(\frac{1}{4}\right)^{\infty} = 3 - 8\frac{1^{\infty}}{4^{\infty}} = 3 -0 = 3[/tex]

--------------------------------------------------

[tex]\lim_{x \rightarrow \infty} g(x) = \lim_{x \rightarrow \infty} 3 - 8\left(\frac{1}{4}\right)^{2-x} = 3 - 8\left(\frac{1}{4}\right)^{2-\infty} = 3 - 8\left(\frac{1}{4}\right)^{-\infty} = 3 - 8\times 4^{\infty} = 3 - \infty = -\infty[/tex]

Thus, the horizontal asymptote is x = 3.

--------------------------------------------------

Question c:

The limit of x going to infinity of the function is negative infinity, which means that the function is decreasing.

--------------------------------------------------

Question d:

Exponential function has no restrictions in the domain, so it is all real values, that is [tex](-\infty,\infty)[/tex].From the limits in item c, the range is: [tex](-\infty,3)[/tex]

--------------------------------------------------

The sketching of the graph is given appended at the end of this answer.

A similar problem is given at https://brainly.com/question/16533631

Apples are cut into 8 pieces to be shared among some children. Twenty-two bags of seven apples are used. How many pieces of apple are cut?

Answers

Answer: 1232 pieces

Work Shown:

1 bag = 7 apples

22 bags = 22*7 = 154 apples

So we have 154 apples to work with in total.

Each of those apples is cut into 8 pieces, giving us 8*154 = 1232 pieces

We can write it as one single calculation to say 22*7*8 = 1232

Find the area of this circle. Use 3 for .
A = r2
12 cm
[?] cm2

Answers

Answer:

452.16 cm²

Step-by-step explanation:

Given :-

Radius = 12cm .

To find :-

Area of circle .

Solution :-

As we know that ,

A = πr² A = 3.14 * (12 cm)² A = 3.14 * 144cm² A = 452.16 cm²

Step-by-step explanation:

[tex]area = \pi {r}^{2} \\ = \pi \times {12}^{2} \\ = \pi \times 144 \\ = 3.14 \times 144 \\ = 452.16 {cm}^{2} \\ thank \: you[/tex]

Solve for x: -5 < 8x + 11 < 19

Answers

Answer:

-2<x<1

Step-by-step explanation:

-5 < 8x + 11 < 19

Subtract 11 from all sides

-5-11 < 8x + 11-11 < 19-11

-16 < 8x<8

Divide by 8

-16/8 < 8x/8 <8/8

-2<x<1

Question 2 Evaluate the expression 2(x - 3) + 3y when x = 5 and y = 3. Mark the correct answer.
A. 13
B. 15
C. 16
D. 25​

Answers

Answer:

A. 13

Step-by-step explanation:

2(x - 3) + 3y

2(5 - 3) + 3×3

2× 2 + 3×3

4 + 9

13

Answer:

The correct answer is A

Step-by-step explanation:

2(x-3) + 3y so you replace them and get 2( 5 - 3) +3(3)

next,

you would solve it

2 x 5 and 2 x 3 you get 10-6 + 9, the 9 is from multiplying 3 by 3

finally you solve it

10-6 = 4 +9

= 13!

9/7 = please answer me​

Answers

Answer:

[tex] \frac{9}{7} = 1.2857 = 1 \frac{2}{7} [/tex]

Answer:

Step-by-step explanation:

[tex]\\\\\\\\\\\\\\\\\\\\\\\\\\\\\\\\\\\\\\\\\\\\\\\\\\\\\\\la\la\la\la\ddddddddddddddddddddddddddddddddcleverdddddd\ffffffffffffffffffffffffffffffffffffffff\pppppppppppppppppppppppppppppppppppp\ddddddddddddddddddd\displaystyle\ \Large \boldsymbol {\frac{9}{7}=1\frac{2}{7} \ \ or \ \ 1.(28571)}[/tex]

Complete the table, and then use the drawing tools to create the graph representing the relationship between the amount of plant food remaining, f(x), and the number of days that have passed, x.

Answers

The complete table of the function is:

[tex]\begin{array}{cccccccc}x & {0} & {1} & {2} & {3} & {4}& {5} & {6} \ \\ f(x) & {72} & {60} & {48} & {36} & {24}& {12} & {0} \ \end{array}[/tex]

The equation missing from the question is:

[tex]y = 72 - 12x[/tex]

To complete the table, we simply calculate the y value for each x value;

When [tex]x = 0[/tex],     [tex]y = 72 - 12 * 0 = 72[/tex]

When [tex]x = 1[/tex],     [tex]y = 72 - 12 * 1 = 60[/tex]

When [tex]x = 2[/tex],     [tex]y = 72 - 12 * 2 = 48[/tex]

When [tex]x = 3[/tex],     [tex]y = 72 - 12 * 3 = 36[/tex]

When [tex]x = 4[/tex],     [tex]y = 72 - 12 * 4 = 24[/tex]

When [tex]x = 5[/tex],     [tex]y = 72 - 12 * 5 = 12[/tex]

When [tex]x = 6[/tex],     [tex]y = 72 - 12 * 6 = 0[/tex]

So, the complete table is:

[tex]\begin{array}{cccccccc}x & {0} & {1} & {2} & {3} & {4}& {5} & {6} \ \\ f(x) & {72} & {60} & {48} & {36} & {24}& {12} & {0} \ \end{array}[/tex]

To create a graph of f(x), we simply type [tex]y = 72 - 12x[/tex] on a drawing tool and the graph will be generated.

See attachment for graph

Learn more about graphs at:

https://brainly.com/question/20106471

Answer:

Step-by-step explanation:


Which number is a solution of the inequality?
3x - 15 ≥ 3

a.) -9/11

b.)6/11

c.)6

d.)5

Answers

Answer:

6

Step-by-step explanation:

[tex]3x - 15 \geqslant 3[/tex]

[tex]3x \geqslant 3 + 15[/tex]

[tex]x \geqslant \frac{18}{3} [/tex]

[tex]x \geqslant 6[/tex]

PLEASE HELP ASAP!!!!! The following graph shows a proportional relationship. What is the constant of proportionality between y and x in the graph?

Answers

Answer:

2/3

Step-by-step explanation:

The graph point is on (3,2)

Find the measure of the indicated angle to the nearest degree.

Answers

Answer:

Step-by-step explanation:

Adjacent to the undetermined angle is 6, and the hypotenuse has been given. We can conclude, after looking at our SOHCAHTOA, that we will be using cosine(CAH) to solve this problem.

Let the unspecified angle be [tex]\theta\\[/tex] (This sign is called theta, which is just a sign for angle)

Lets start!

cos[tex]\theta\\[/tex] = adj/hyp

cos[tex]\theta\\[/tex] = 6/13

[tex]\theta\\[/tex] = [tex]cos^{-1}[/tex](6/13)

[tex]\theta\\[/tex] = 62.5

[tex]\theta\\[/tex] = 63

Hope that helped!

Write the equation of the line that passes through the points (−7,5) and (-7,-8).
Put your answer in fully reduced point-slope form, unless it is a vertical or horizontal line.

Answers

Answer:

x = -7

Step-by-step explanation:

First we find the slope using

m = ( y2-y1)/(x2-x1)

   = ( -8 - 5)/( -7 - -7)

   = (-8-5)/(-7+7)

   = -13/0

This means the slope is undefined and the line is vertical

Vertical lines are in the form

x= constant and the constant is the x value of the points

x = -7

If P =
1 2
then, prove that P2 - 2P - 5 = 0, 1 = 0, where I and 0 are unit
3 1
matrix and null matrix of order 2 x 2 respectively.

Answers

Answer:

thanks for free ponits you dint gave it free but I theft I'm sorry

Which shapes have the greatest area?
(Select all that apply.)

Answers

Answer:

the trapiezium

Step-by-step explanation:

It has a area of 4 which is the highest

(x^2+1)(x-1)=0 help me pls

Answers

Answer:

x = ±i ,  x=1

Step-by-step explanation:

(x^2+1)(x-1)=0

Using the zero product property

x^2 +1 = 0   x-1= 0

x^2 = -1       x=1

Taking the square root of the equation on the left

sqrt(x^2) = sqrt(-1)

x = ±i   where  i is the imaginary number

We still have x=1 from the equation on the right

If the lengths of the legs of a right triangle are 4 and 8, what is the length of the hypotenuse?

PLEASE HELP

Answers

Answer:

[tex]4\sqrt{5}[/tex]

Step-by-step explanation:

In order to solve this problem, we can use the pythagorean theorem, which is

a^2 + b^2 = c^2, where and b are the legs of a right triangle and c is the hypotenuse. Since we are given the leg lengths, we can substitute them in. So, where a is we can put in a 4 and where b is we can put in an 8:

a^2 + b^2 = c^2

(4)^2 + (8)^2 = c^2

Now, we can simplify and solve for c:

16 + 64 = c^2

80 = c^2

c = [tex]\sqrt{80}[/tex]

Our answer is not in simplified radical form because the number under is divisible by a perfect square, 16. We can divide the inside, 80,  by 16, and add a 4 on the outside, as it is the square root of 16:

c = [tex]4\sqrt{5}[/tex]

The length of the hypotenuse in the given right triangle, with legs measuring 4 and 8, is approximately 8.94.

To find the length of the hypotenuse in a right triangle, we can use the Pythagorean theorem. According to the theorem, in a right triangle, the square of the hypotenuse's length is equal to the sum of the squares of the lengths of the other two sides.

In this case, let's label the lengths of the legs as 'a' and 'b', with 'a' being 4 and 'b' being 8. The hypotenuse, which we need to find, can be represented as 'c'.

Applying the Pythagorean theorem, we have:

[tex]a^2 + b^2 = c^2[/tex]

Substituting the given values:

[tex]4^2 + 8^2 = c^2[/tex]

16 + 64 = [tex]c^2[/tex]

80 = [tex]c^2[/tex]

To find the length of the hypotenuse 'c', we need to take the square root of both sides:

√80 = √ [tex]c^2[/tex]

√80 = c

The square root of 80 is approximately 8.94.

Therefore, the length of the hypotenuse in the given right triangle, with legs measuring 4 and 8, is approximately 8.94.

To know more about  hypotenuse , here

https://brainly.com/question/2217700

#SPJ2

The path from the subway station to the art museum is three blocks to the north then four blocks to the west.
What is the straight-line distance in blocks from the subway station to the art museum?

Answers

Answer:

7 block s

Step-by-step explanation:

3 from North plus

4 from West

4+3=7

Tyrone measured the floor of his rectangular storage unit. It is 3 feet wide and 8 feet from one corner to the opposite corner. How long is the storage unit? If necessary, round to the nearest tenth.

Answers

Answer:

Rounded to the nearest tenth, 7.4 feet long.

Step-by-step explanation:

Tyrone has a rectangular storage unit. We are given the width and the diagonal length.

So we can use Pythagorean Theorem.

3^2 + b^2 = 8^2

9 + b^2 = 64

subtract 9 from both sides

b^2 = 55

b = sqrt55

b is around 7.4161984871, so b rounded to the nearest tenth is 7.4 feet long.

The distance AB rounded to the nearest tenth = [?]

Answers

Answer:

4.5 units

Step-by-step explanation:

Use the distance formula

[tex]\sqrt{(-1-3)^{2}+(-1-1)^{2} }[/tex]

[tex]\sqrt{16+4}=\sqrt{20}[/tex]

The distance AB on the diagram rounded to the nearest tenth is: 4.5 units

Meaning of Distance

Distance can me defined as a measure that tells us how far apart two objects or individual are to each other.

Distance is very important as it helps us know where exactly things are located and whether they are close or  far apart

In conclusion, The distance AB on the diagram rounded to the nearest tenth is: 4.5 units

Learn more about Distance: https://brainly.com/question/17273444

#SPJ2

use the prime factors of 3136 and 2744 to evaluate:✓3136/3✓2744​

Answers

Answer:

3136 = 2^6 × 7^2

2744 = 2^3 x 7^3

✓3136/3✓2744​ = ✓(2^6 × 7^2)/3✓(2^3 x 7^3) = (2^3 x 7)/3 x 14(✓14) = 56/42✓14 =4/3✓14

Find the length of UC? Please help

Answers

Answer:

The choose C. 18

Step-by-step explanation:

UC —> 105+82=187 —> 96+22+51=169 —> 187–169=18

I hope I helped you^_^

explanation would be appreciated. i don’t understand

Answers

Answer:

[tex]28\sqrt{3}[/tex]

Step-by-step explanation:

The area of the big triangle is 1/2 b h =  1/2*6*(12^2 = 6^2 + x^2)

that ends up being [tex]\sqrt{108} = 36\sqrt{3}[/tex]

the small triangle are needs to be subtracted....

[tex]\frac{\left(4\cdot \:sin\left(90\right)\right)}{sin\left(30\right)}[/tex] that is the length of the unknown side...

1/2 B * h of that triangle get you to [tex]8\sqrt{3}[/tex]

just subtract the two areas

Answer:

(B) 28√3

Step-by-step explanation:

The area of quadrilateral ABED is equal to the area of triangle CDE subtracted from the area of triangle ABC.

Area of triangle CDE:

Triangle ABC is equilateral. All sides have length 12.

AB = BC = AC = 12

BE = 8

BE + EC = BC

8 + EC = 12

EC = 4

In an equilateral triangle, all angles measure 60°.

m<C = 60°

m<CDE = 30°

Triangle CDE is a 30-60-90 triangle.

DE = EC√3

DE = 4√3

area of triangle CDE = bh/2

area of triangle CDE = (EC)(DE)/2

area of triangle CDE = (4)(4√3)/2

area of triangle CDE = 8√3

Area of triangle ABC:

Side AC is a base of triangle ABC.

AC = 12

(1/2)AC = 6

The altitude of triangle ABC from side AC to vertex B measures

h = 6√3

area of triangle ABC = bh/2

area of triangle ABC = (AC)(h)/2

area of triangle ABC = (12)(6√3)/2

area of triangle ABC = 36√3

area of quadrilateral ABED = area of triangle ABC - area of triangle CDE

area of quadrilateral ABED = 36√3 - 8√3

area of quadrilateral ABED = 28√3

What is the estimate for 312+138+207

Answers

Answer:

657

Step-by-step explanation:

312 + 138 + 207

= 450 + 207

= 657

Answer:

600

Step-by-step explanation:

To round your answer, you check the tenths place to see if it is under 5 or above 5. If it is under 5, your answer will stay in the number the hundreds place is in currently. If it is above 5, you will add one to the hundreds place.

312: there is one in the tenths place, so it will stay as 300.

138: there is three in the tenths place, so it will stay as 100.

207: there is a zero in the tenths place, so it will stay as 200.

If you are doing it with the one's place, it is the same method. Either round up or down.

300 + 100 + 200 = 600

The answer is 600.

Find the 5th term of each geometric sequence. 32,80, 200

Answers

Answer:

12.8

Step-by-step explanation:

Which equation can be solved using the one-to-one property?
3X = 10
4In x = 2
log x = 5
4* = 47x+2

Answers

Answer:

3x=10

Step-by-step explanation:

x=10-3

x=7

i hope this answer will help u

Answer:

4x = 47x+2

Step-by-step explanation:

Using the one–to–one property, you can set x = 7x + 2.

what is the area of the figure below?

Answers

Answer:

15x^9

Step-by-step explanation:

A=l x w

5x^4 times 3x^5 is basically 3*5*x^4*x^5

when you multiply exponents with the same base, you add the exponents, so it becomes 15x^9

Other Questions
evaluate (-1)^6-4^0+(3/7)^0 A ball is dropped from the roof of a 25-m-tall building. What is the velocity of the object when it touches the ground? Suppose the ball is a perfect golf ball and it bounces such that the ve locity as it leaves the ground has the same magnitude but the op posite direction as the velocity with which it reached the ground How high will the ball bounce? Now suppose instead that the ball bounces back to a height of 20 m. What was the velocity with which it left the ground? what is the value of x What volume of 1.50 mol/L stock solution is needed to make 125 mL of 0.60 mol/L solution? Setting y = 0 allows you to determine the what of a graph Suppose the distributor charges the artist a $40.00 cost for distribution, and the streaming services pays $4.00 per unit. (Note: One unit = one thousand streams)- - - - - - - - - - - - - - - - - - - - - - - - - - - - - - - - - - - - - - - - - - - - - - - - - - - - - - - - - - - -Formula: y = 40x + 4 (Graph Attached)- - - - - - - - - - - - - - - - - - - - - - - - - - - - - - - - - - - - - - - - - - - - - - - - - - - - - - - - - - - -After how many streams will you pay for the distributor charges? (Hint: this is where the line crosses the x-axis, round to the nearest thousand) In A Journey to the Center of the Earth if the Theory or Central Fire were true, what would be happening?The humans would become roasted very quickly.The rocks should become more solid as they get closer to the center of the Earth.The temperature should be increasing as they descend into Earth.The temperature should be decreasing as they descend into Earth. Which choice correctly shows the solution(s) of the equation x2 = 1442A)x= 144B)x=V12X=--V144D)x = 1V144 trnh by cc vai tr ca o c kinh doanh trong pht trin doanh nghip Why can viruses not fit in any domain? How are they different from all other forms of Life? What would need to occur for viruses to be classified as "alive"? Please help im begging youFind the domain of the function expressed by the formula:y = 1/x - 7 What is the volume of a metal block 3cm long by 2cm wide by 4cm high? What would be the volume of a block twice as long, wide, and high? John earns $6 per hour for mowing the lawn. If t represents John's total earnings for h hours of mowing, which equations can be used to model the situation However burdensome their lives from sunup to sundown, after work slaves established a sense of self-worth and community. Discuss the factors that made this possible. Answer in 3-5 paragraphs if possible thank you! Why was a weak national government created under the Articles of Confederation? "A parabola has the equation = ^ + . What are the coordinates of the vertex? (You must solve by factoring)!!!!!" I NEED THE ANSWER TO THIS FAST WITH STEPS I'm a grade 10 academic student by the way Part of the president job involves foreign affaires which include working with ambassadors to foreign countries how do these ambassadors get their jobs If f(x) = 4x + 5 and fog(x) = 8x + 13 then find g(x). What is 4,327 rounded to the nearest thousand? 12. Convert 30.283 into a degree-minute-second format.O A. 18 16' 98"B. 18 28' 30"C. 30 16' 58"D. 30 28' 30"